Complex contour integral of square-root

Click For Summary
To evaluate the contour integral of \( z^{1/2}dz \) around the specified closed contour C, it is essential to understand the implications of the branches for the square root function. The branches dictate which square root to use for each segment of the contour: C1 above the x-axis uses the branch where \( -\pi/2 < \theta < 3\pi/2 \), while C2 below the x-axis uses the branch where \( \pi/2 < \theta < 5\pi/2 \). The integral can be computed by taking the anti-derivative \( \frac{2}{3}z^{3/2} \) and integrating along each contour segment separately. Careful attention to the chosen branch for each segment is crucial to avoid errors in the integration process. Understanding these branches is key to successfully evaluating the integral.
Yeggoua
Messages
1
Reaction score
0
For my homework I am told: "Evaluate $z^(1/2)dz around the indicated not necessarily circular closed contour C = C1+C2. (C1 is above the x axis, C2 below, both passing counter-clockwise and through the points (3,0) and (-3,0)). Use the branch r>0, -pi/2 < theta < 3*pi/2 for C1, and the branch r>0, pi/2 < theta < 5*pi/2 for C2."

I am unsure how to approach this and would appreciate help. I get that I need to take the anti-derivative to get 2/3*z^(3/2) and possibly convert to polar form, but am unsure how to handle the different 'branches' to find an answer.

Thanks
 
Physics news on Phys.org
Yeggoua said:
For my homework I am told: "Evaluate $z^(1/2)dz around the indicated not necessarily circular closed contour C = C1+C2. (C1 is above the x axis, C2 below, both passing counter-clockwise and through the points (3,0) and (-3,0)). Use the branch r>0, -pi/2 < theta < 3*pi/2 for C1, and the branch r>0, pi/2 < theta < 5*pi/2 for C2."

I am unsure how to approach this and would appreciate help. I get that I need to take the anti-derivative to get 2/3*z^(3/2) and possibly convert to polar form, but am unsure how to handle the different 'branches' to find an answer.

The bit about the branches is that you need to choose which square root you use when you're integrating. (Recall that every number has two square roots.) That's really all that the 'branches' stuff is about.

You should be able to integrate along the two pieces seperately without any special problems.
 
Question: A clock's minute hand has length 4 and its hour hand has length 3. What is the distance between the tips at the moment when it is increasing most rapidly?(Putnam Exam Question) Answer: Making assumption that both the hands moves at constant angular velocities, the answer is ## \sqrt{7} .## But don't you think this assumption is somewhat doubtful and wrong?

Similar threads

  • · Replies 3 ·
Replies
3
Views
2K
Replies
1
Views
1K
  • · Replies 8 ·
Replies
8
Views
2K
  • · Replies 19 ·
Replies
19
Views
2K
Replies
32
Views
3K
Replies
9
Views
2K
  • · Replies 3 ·
Replies
3
Views
3K
  • · Replies 9 ·
Replies
9
Views
2K
  • · Replies 2 ·
Replies
2
Views
2K
  • · Replies 11 ·
Replies
11
Views
4K